数字信号处理习题答案

更新时间:2023-12-28 07:28:01 阅读量: 教育文库 文档下载

说明:文章内容仅供预览,部分内容可能不全。下载后的文档,内容与下面显示的完全一致。下载之前请确认下面内容是否您想要的,是否完整无缺。

部分练习题参考答案

第二章

2.1 x(n)?2?(n?2)??(n?1)?3?(n)?2?(n?1)

??(n?2)??(n?3)?2?(n?4)??(n?6) 2.2 其卷积过程如下图所示

x(m) 2 1 0.5

m

0 h(0-m)

2

1 0

-1

y(n)

h(1-m) 2

1

m

2

2.5

m

-1

5

0

-1 1

m

h(m) 2

h(-1-m)

2 1

m

0

n

0

-1

-1 -0.5

0

-1

372?143y(n)?2?(n)?5?(n?1)?2.5?(n?2)??(n?3)??(n?4)?0.5?(n?5)

2.3 (1)??(2)p??,??这是有理数,因此是周期序列。周期N=14。

?16?k,k取任何整数时,p都不为整数,因此为非周期序列。 1/82?k122?k?k,p2??4k,当p1,p2 同时为整数时k=5,x(n)为周期序(3)p1?5/6?50.5?2?k列,周期N=60。

(4)p?2?k1.6??1.25?k,取k=4,得到p=6,因此是周期序列。周期N=6。

?2.4 (1) y(n)?x(n)?h(n)?(a) 当n<0 时,y(n)=0

?Rm???5(m)R4(n?m)

(b) 当0?n?3时,y(n)? (c) 当4?n?7时,y(n)? (d) 当n>7时,y(n)=0

n?1?n?1

m?03?1?8?n

m?n?4n?0或n?7?0?0?n?3 所以y(n)??n?1?8?n4?n?7?(2)y(n)?2R4(n)?[?(n)??(n?2)]?2R4(n)?2R4(n?2)

?2[?(n)??(n?1)??(n?4)??(n?5)]

?n?m(3)y(n)?x(n)?y(n)??Rm???n5(m)0.5u(n?m)

?m??0.5?Rm???(m)0.55u(n?m)

(a) 当n<0 时,y(n)=0

n(b) 当0?n?4时,y(n)?0.54n?m?00.5?m?0.5nn1?25n?11?2?2?0.5

nn(c) 当n?5时,y(n)?0.5n?0.5m?0?m?0.51?21?2?31?0.5

nn最后写成统一表达式:y(n)?(2?0.5)R5(n)?31?0.5u(n?5)

?(4)y(n)?x(n)?h(n)?(a) 当n?0 时,y(n)=0

?Rm???3(m)0.5n?m

n?1 (b) 当1?n?3时,y(n)?0.5 (c) 当4?n?5时,y(n)?0.5 (d) 当n?6时,y(n)=0

n?0.5m?03?m?0.5n1?22n1?2n?2n?1?0.5 (1?21?26?nnn?0.5?m?0.5)?16?0.5?0.25

nm?n?2y(n)?0.5?(n?1)?0.75?(n?2)?0.875?(n?3)?0.75?(n?4)?0.25?(n?5)

2.6 (1)非线性、移不变系统

(2)线性、移不变系统

(3)线性、移变系统

(4)非线性、移不变系统 (5)线性、移变系统

2.7 (1)若g(n)??,则稳定,因果,线性,时变

(2)不稳定,n?n0时因果,n?n0时非因果,线性,时不变 (3)线性,时变,因果,不稳定 2.8 (1)因果,不稳定

(2)因果,稳定

(3)因果,稳定 (4)因果,稳定 (5)因果,不稳定 (6)非因果,稳定 (7)因果,稳定 (8)非因果,不稳定 (9)非因果,稳定

(10)因果,稳定

2.9 因为系统是因果的,所以n?0,h(n)?0

令x(n)??(n),y(n)?h(n)?0.5h(n?1)?x(n)?0.5x(n?1) h(0)?0.5h(?1)?x(0)?0.5x(?1)?1

h(1)?0.5h(0)?x(1)?0.5x(0)?0.5?0.5?1 h(2)?0.5h(1)?x(2)?0.5x(1)?0.5 h(3)?0.5h(2)?x(3)?0.5x(2)?0.5 h(n)?0.5h(n?1)?x(n)?0.5x(n?1)?0.5n?12

所以系统的单位脉冲响应为h(n)??(n)?0.5n?1u(n?1) 2.10 (1)初始条件为n<0时,y(n)=0

设x(n)??(n),输出y(n)就是h(n)

上式可变为

h(n)?0.5h(n?1)??(n)

可得 h(0)?0.5h(?1)?1?1

依次迭代求得h(1)?0.5h(0)?0?0.5

h(2)?0.5h(1)?0?0.5

2?

h(n)?0.5h(n?1)?0?0.5

n故系统的单位脉冲响应为h(n)?0.5nu(n) (2)初始条件为n≥0时,y(n)=0

y(n?1)?2[y(n)?x(n)]

h(n)?0,n?0

2h(?1)?2[h(0)?x(0)]??2 h(?2)?2[h(?1)?x(?1)]??2 h(?3)?2[h(?2)?x(?2)]??2

3?

h(n)?2h(n?1)??2

n所以h(n)??2u(?n?1) 2.11 证明

?n(1)因为x(n)?h(n)?令m'?n?m,则

?x(m)h(n?m)

m????x(n)?h(n)??x(n?m')h(m')?h(n)?x(n)

m'???(2)利用(1)证明的结果有

x(n)?[h1(n)?h2(n)]?x(n)?[h2(n)?h1(n)]

????x(m)[hm????2(n?m)?h1(n?m)] (k)h1(n?m?k)

??m????x(m)?hk????2交换求和的次序有

x(n)?[h1(n)?h2(n)]???k????h2(k)?x(m)hm???1(n?m?k)

?hk???2(k)[x(n?k)?h1(n?k)]

?h2(n)?[x(n)?h1(n)] ?[x(n)?h1(n)]?h2(n)

?(3)x(n)?[h1(n)?h2(n)]???x(m)[hm????1(n?m)?h2(n?m)]

??m???x(m)h1(n?m)??x(m)hm???2(n?m)

?x(n)?h1(n)?x(n)?h2(n)

?2.12 y(n)?x(n)?y(n)??Rm???N(m)a?n?mu(n?m)

?m?an?Rm???N(m)au(n?m)

(a) 当n<0 时,y(n)=0

n(b) 当0?n?N?1时,y(n)?aN?1n?m?0an?m?an1?(1/a)Nn?11?1/a?an?1??aan?1?1a?1n?N?1

(c) 当n?N时,y(n)?a最后写成统一表达式:y(n)?an?am?0?m?a1?(1/a)1?(1/a)a?1

n?1RN(n)?a?1a?12.13 y1(n)?x(n)?h1(n)?u(n)*[?(n)??(n?4)]

?1an?1?an?N?1u(n?N)

?u(n)?u(n?4)?R4(n)

y(n)?y1(n)?h2(n)?R4(n)?au(n)

na?12.14 (1)采样间隔为T?1/200?0.005

??an?1?1R4(n)?an?1?an?3a?1u(n?4)

?a(t)?x??n????sin(2?f0nT??88)?(t?nT)

?n???sin(100?nT???8)?(t?nT) )

(2)x(n)?sin(0.5?n?数字频率??0.5?,

x(n)

2???4,周期N=4

0.92

0.38

n

0

-0.38

-0.92

?

?j?n2.15 (1)X(ej?)???(n?n0)en?????e?j?n0

??j?n (2)X(ej?)??x(n)en????????en?0(???j?0n)e?j?n

?e?en?0j(?0??)n

?e1?e???j(?0??)

????n (3)X(ej?)???x(n)en????j?n??en?0e?j?n??en?0?(??j?)n

11?e??(??j?)

? (4)X(ej?)???x(n)en?????j?n??en?0??ncos?0ne)e?j?n?j?n

(???j?0?j?)n?n?0e??n12(ej?0n?e?j?0n?12??[en?0?e??(???j?0?j?)n]

1?1??2?1?e??e?j(?0??)?11?e2N?1??e?j(?0??)1?eecos?0???1?2e?j?e??cos??e?2j?e?2? ?0?j?(5)X(e2N?1j?)?12?n???2N?1x(n)ej?j?n??j??????j?n1?cosn??e ????N??n?N?)e?j?n?N??n?Ne?j?n??n?N(en?N?ej(n

?N??)N?j(?e?j?N(1?e?j?N1?e?j??)1?e?2????N??)N(1?ej(j(?N)?N?e??)N(1?e?j(?j(?N??)N1?e??)?N1?e??)?)? ???

sin(??2N)e?j(32N?1)?1?cos?1?2cos?Nee?j?sin?2?Ncos(?e12??j2??2N)e?j32N?

?j?2.16 (1)h(n)?12?????H(enj?)ej?nd?????0jej?nd??12???0jej?nd?

?1?(?1)n??0???2??n?n为偶数n为奇数

(2)y1(n)?x1(n)?h(n)?sin(?0n)

y2(n)?x2(n)?h(n)??cos(?0n)

2.17 (1)X?(e?j?)

(2)(3)

1212[X(ejj?)?X(ej??j?)]

?2?2[X(e)?X(?e)]

(4)X(e2j?)

2.18采样间隔为T?0.25,采样频率?s?8?

ya1(t)没有失真,因为输入信号的频率?1?2?小于

?s2?4?

ya2(t)失真,因为输入信号频率?2?5?大于

?s2?4?

第三章

3.1 设X(ej?)和Y(ej?试求下列序列的傅里叶变换: )分别是x(n)和y(n)的傅里叶变换,

*(1)x(n?n0) (2) x(n)

(3) x(?n) (4) x(n)*y(n) (5) x(n)?y(n) (6) nx(n) (7) x(2n) (8)x(n) ?x(n2),n?偶数(9)x9(n)??

n?奇数?0,2?解:(1) FT[x(n?n0)]=

?x(n?nn???0)e?j?n

令n??n?n0,n?n??n0,则

?FT[x(n?n0)]=

??x(n?)en???*?j?(n???n0)?e?j?n0X(ej?)

?(2) FT[x(n)]=

*?xn????(n)e?j?n?[?x(n)en????j?n]?X*(e*?j?)

(3) FT[x(?n)]=令n???n,则

?x(?n)en????j?n

?FT[x(?n)]=

?x(n?)en???j?n??X(e?j?)

(4) FT[x(n)*y(n)]=X(ej?)Y(ej?)

?证明 x(n)*y(n)=

?x(m)y(n?m)

m??????j?nFT[x(n)*y(n)]=

令k?n?m,则

?[?x(m)y(n?m)]en???m???

???j?kFT[x(n)*y(n)]=

?[?x(m)y(k)]ek???m?????e?j?m

=

?k???y(k)e?j?k?x(m)em????j?m

=X(e?j?)Y(ej?)

(5) FT[x(n)?y(n)] =

?x(n)y(n)en?????j?n

=

?n???x(n)[12???Y(e???j??)ej??nd??]e?j?n

=

12?????Y(ej??)?n???x(n)e?j(????)nd??

=

或者 FT[x(n)y(n)]=

j?12???Y(e?j??j??)X(ej?j(????))d??

12??X(e)*Y(e)

(6) 因为X(e)??x(n)en????j?n,对该式两边对?求导,得到

dX(ed?j?)???j?nx(n)en???j??j?n??jFT[nx(n)]

因此 FT[nx(n)]=jdX(ed??)

(7) FT[x(2n)]=

令n??2n,则

FT[x(2n)]=

?x(2n)en????j?n

?x(n?)en?取偶数??j?n?2

=

?n???12?[x(n)?(?1)x(n)]en1?j?n2

1?j?n2 =[21?x(n)en???1j?21?j?n2???en???j?nx(n)e]

=[X(e21)?X(e1j(???)2)]

或者

FT[x(2n)]=

12?[X(e1j?2)?X(e1j?2)]?X(e1j?2)

(8) FT[x(n)]=

2?xn???2(n)e?j?n

利用(5)题结果,令x(n)?y(n),则

FT[x(n)]=

212?X(ej?)*X(e?j?)=

12???X(e??j??)X(ej(????))d??

(9) FT[x9(n)]=

?n???n取偶数n?j?n x()e2令n??n2,???n???,则

?FT[x9(n)]=

3.2 已知

X(ej??x(n?)en???n取偶数?j2?n??X(ej2?)

|?|??0?1, )??0,??|?|??0?求X(ej?)的傅里叶反变换x(n)。

12??0??0j?n解: x(n)??ed??sin?0n?n

3.3

线性时不变系统的频率响应(传输函数)H(ej?)?|H(ej?)|ej?(?),如果单位脉冲响应h(n)为实序列,试证明x(n)?Acos(?0n??)的稳态响应为

y(n)?A|H(ej?0)|cos[?0n????(?0)]

解:假设输入信号x(n)?ej?0n,系统单位脉冲响应为h(n),系统输出为

?j?0(n?m)j?0n?y(n)?x(n)*h(n)??h(m)em????e?h(m)em????j?0m?H(ej?0)ej?0n

上式说明,当输入信号为复指数序列时,输出序列仍是复指数序列,且频率相同,但幅度和

相位决定于网络传输函数,利用该性质解此题。

x(n)?Acos(?0n??)=y(n)?1212A[eA[ej?0n12j?0A[ej?0nej??e?j??j?0ne?j?]

ej?H(e)?e)|e?j?0neH(e?j?0)] |H(e?j?0 =上式中|H(ej?j?0nej?|H(ej?0j?(?0)?e?j?0ne?j?)|ej?(??0)]

)|是?的偶函数,相位函数是?的奇函数,即

|H(ej?0)|=|H(e12A|H(ej?0?j?0)|, ?(?0)???(??0)

j?0ny(n)?j?0)|[eej?ej?(?0)?e?j?0ne?j?e?j?(?0)]

=A|H(e3.4 设

)|cos[?0n????(?0)]

?1,x(n)???0,n?0,1其它

将x(n)以4为周期进行周期延拓,形成周期序列~画出x(n)和~求出~x(n),x(n)的波形,x(n)的离散傅里叶级数X(k)和傅里叶变换。 解:画出x(n)和~x(n)的波形如题4解图所示。

~

题4解图

~X(k)?DFS[~x(n)]??43?n?0?j?j~x(n)e2?4kn1??n?0e?j?2kn?1?e?j?2k

=e?jk??e??j?4k?4?ek??jk??~??2cos(k)?e4,X(k)以4为周期 ?4??X(ej?2?)?FT[~x(n)]?42?~X(k)?(??k) ?4k???=

??~X(k)?(??k) ?2k???2??=??k???cos(?4k)e?j?4k?(???2k)

3.5 设题5图所示的序列x(n)的FT用X(e(1)X(e(2)??j0j?不直接求出X(e)表示,

j?完成下列运算: ),

);

j???X(ej?)d?;

(3)X(e);

(4)确定并画出傅里叶变换实部

Re[X(e?j?)]的时间序列xe(n);

j?(5)?|X(e??)|d?;

j?2(6)?|???dX(ed?)2|d?。

7解:(1) X(e?j0)=

?x(n)?6

n??3(2)

???X(ej?j?)d?=x(0)?2?=4?

?7?j?nn(3) X(e)=

?x(n)en?????(?1)n??3x(n)?2

(4)因为傅里叶变换的实部对应序列的共轭对称部分,即

?Re[X(ej?)]=

?xn???e(n)e?j?n

xe(n)?12(x(n)?x(?n))

按照上式画出xe(n)的波形如题5解图所示:

(5)?|X(e???j?7)|d?=2?2?|x(n)|n??32?28?

(6)因为

dX(ed??j?)?FT[?jnx(n)]

因此

??|?dX(ed?j?)27|d?=2??|nx(n)|n??32?316?

3.6 试求如下序列的傅里叶变换: (1)x1(n)??(n?3) (2)x2(n)?12?(n?1)??(n)?12?(n?1)

n(3)x3(n)?au(n),0?a?1

(4)x4(n)?u(n?3)?u(n?4) 解:

?(1)X1(ej?)???(n?3)en?????j?n?e?j3?

(2)X2(ej?)??n????x2(n)e?j?n?12ej??1?12e?j??1?12(ej??e?j?)?1?cos?

?(3)X3(ej?)??n????au(n)en?j?n??n?0aen?j?n?11?ae3?j?

(4)X4(ej?)??[u(n?3)?u(n?4)]en???3?1?j?n3?j?n?j?n??en??33?j?n

=?en?0??en??3??en?0?j?n??en?1j?n

?1?e?j4??j?1?eej3??1?e1?ej3?j?ej??1?e?j4??j?1?e?1?e1?ej3??j?

=

?e?j4?1?e7?j?2?j?21?j??1?e1?e7?j?2?j?21j7??j?ej3?

7212=

ee(e(e7j?2j?21?e?e))sin(ej3??)

?sin(?)或者 x4(n)?u(n?3)?u(n?4)=R7(n?3)

?j?X4(e)??Rn???7(n?3)e?j?n

6FT[R7(n)]???en?0?j?n?1?e?j7??j?1?e?j?n

j7??j?X4(ej?)??Rn???7(n?3)e?1?e1?eej3?

7j?2j?217?j?2?j?21=

ee7?j?2?j?21(e(e7j?2j?21?e?e7?j?2?j?21))ej3??ee1?j?2?j?21(e(e?e?e))sin(?sin(7212?)

?)3.7 设:

(1)x(n)是实偶函数, (2)x(n)是实奇函数,

分别分析推导以上两种假设下,其x(n)的傅里叶变换性质。 解:令

? X(ej?)??x(n)en????j?n

(1)x(n)是实偶函数,

?X(ej?)??x(n)en????j?n

两边取共轭,得到

??j?nX(e*j?)??x(n)en?????x(n)en????j(??)n?X(e?j?)

因此

X(ej?)?X(e*?j?)

上式说明x(n)是实序列,X(ej?)具有共轭对称性质。

???j?nj?X(e)??x(n)en?????x(n)[cos??n???jsin?]

由于x(n)是偶函数,x(n)sin?是奇函数,那么

??x(n)sin?n????0

因此

?X(ej?)??x(n)cos?

n???该式说明X(ej?)是实函数,且是?的偶函数。

j?总结以上x(n)是实偶函数时,对应的傅里叶变换X(e(2)x(n)是实奇函数,

上面已经推出,由于x(n)是实序列,X(eX(e?j?j?)也是实偶函数。

)具有共轭对称性质,即

*?j?)?X(e?)

X(ej?)??x(n)en????j?n??x(n)[cos??n???jsin?]

由于x(n)是奇函数,x(n)cos?是奇函数,那么

??x(n)cos?n????0

因此

X(ej?)?j?x(n)sin?

n????该式说明X(ej?)是纯虚数,且是?的奇函数。

3.8 设x(n)?R4(n),试求x(n)的共轭对称序列xe(n)和共轭反对称序列xo(n),并分别用图表示。 解: xe(n)?12(R4(n)?R4(?n)),xo(n)?12(R4(n)?R4(?n))

xe(n)和xo(n)的波形如题8解图所示。

题8解图

3.9设x(n)?au(n),变换。

n0?a?1,分别求出x(n)的偶函数xe(n)和奇函数xo(n)的傅里叶

?解: X(ej?)??x(n)en???j??j?n

j?因为xe(n)的傅里叶变换对应X(ej,因此

FT[xe(n)]=Re[X(e=

j?)的实部,xo(n)的傅里叶变换对应X(e)的虚部乘以

)]=Re[11?ae?j?]?Re[11?ae?j??1?ae1?ae?j??j?]

1?acos?1?a2?2acos?j?

11?ae?j?FT[xo(n)]=jIm[X(e)]=jIm[]?jIm[11?ae?j??1?ae1?ae?j??j?]

=

?asin?1?a2?2acos?

3.10 若序列h(n)是实因果序列,其傅里叶变换的实部如下式:

HR(ej?)?1?cos?

求序列h(n)及其傅里叶变换H(ej?)。

?解: HR(ej?)?1?cos??1?ej??e?j??FT[he(n)]??hn???e(n)e?j?n

?1?2,n??1?he(n)??1,n?0

?1?,n?1?2?0,n?0??1,n?0???h(n)??he(n),n?0???1,n?1

?2h(n),n?0??0,其它n?e???H(ej?)??n???h(n)e?j?n?1?e?j??2e?j?2cos?2

3.11 若序列h(n)是实因果序列,h(0)?1,其傅里叶变换的虚部为

HI(ej?)??sin?

求序列h(n)及其傅里叶变换H(e解: HI(ej?)??sin???j?)。

j??j?12j(e?e)

FT[ho(n)]?jHI(e????ho(n)?????1j?)??12?(ej??e?j?)??hn???o(n)e?j?n

20,n?01,n?12,n??1

?0,n?0??1,n?0???h(n)??h(n),n?0???1,n?1

?2h(n),n?0??0,其它n?o??

?H(ej?)??n???h(n)e?j?n?1?e?j??2e?j?2cos?2

3.12 设系统的单位取样响应h(n)?anu(n),0?a?1,输入序列为

x(n)??(n)?2?(n?2)

完成下列各题:

(1) 求出系统输出序列y(n);

(2) 分别求出x(n)、h(n)和y(n)的傅里叶变换。 解:

(1)y(n)?h(n)*x(n)?anu(n)*[?(n)?2?(n?2)]=anu(n)+2an?2u(n?2)

?(2) X(ej?)??[?(n)?2?(n?2)]en??????j?n?1?2e?j2?

H(ej?)??n???au(n)en?j?n??n?0aen?j?n?11?ae?j?

Y(ej?)?H(ej?)?X(ej?)?1?2e1?ae?j2??j?

3.13已知xa(t)?2cos(2?f0t),式中f0?100Hz,以采样频率fs?400Hz对xa(t)进行

?a(t)和时域离散信号x(n),试完成下面各题: 采样,得到采样信号x(1)写出xa(t)的傅里叶变换表达式Xa(j?);

?a(t)和x(n)的表达式; (2)写出x?a(t)的傅里叶变换和x(n)的傅里叶变换。 (3)分别求出x解:

(1)Xa(j?)?????xa(t)e?j?tdt?????2cos(?0t)e?j?tdt?????(ej?0t?e?j?0t)e?j?tdt

上式中指数函数的傅里叶变换不存在,引入奇异函数?函数,它的傅里叶变换可表示成:

Xa(j?)?2?[?(???0)??(???0)]

??a?a(t)?(2) x?xn???(t)?(t?nT)??2cos(?n???0nT)?(t?nT)

x(n)?2cos(?0nT),???n??

?0?2?f0?200?rad,T?1fs?2.5ms

?(j?)?1(3) XaT2?T??Xr????a(j??jr?s)

=

?[?(???r???0?r?s)??(???0?r?s)]

式中 ?s=2?fs?800?rads

???j?n?0j?X(e)??x(n)en?????2cos(?n???nT)e?j?n??2cos(?n???0n)e?j?n

??j?0n=

?[en????e?j?0n]e?j?n?2??[?(???k???0?2k?)??(???0?2k?)]

式中 ?0??0T?0.5?rad

上式推导过程中,指数序列的傅里叶变换仍然不存在,只有引入奇异函数?函数,才能写出它的傅里叶变换表示式。

3.14 求出以下序列的Z变换及收敛域。 (1)2?nu(n) (2)?2?nu(?n?1) (3)2?nu(?n) (4)?(n) (5)?(n?1) (6)2解:

???n[u(n)?u(n?10)]

(1)ZT[2?nu(n)]??n???2?nu(n)z?n??n?02?nz?n?11?2???1z?1,|z|?12

?(2)ZT[?2?nu(?n?1)]???2n?????nu(?n?1)z?n???2n??1?nz?n

??n?1?n?2znn??2z1?2z???11?2?n?1z?1,|z|?12

?(3)ZT[2?nu(?n)]??2n???u(?n)z?n??2n?0?nz

???2n?0nzn?11?2z,|z|?12

(4)ZT[?(n)]=1,0?|z|?? (5)ZT[?(n?1)]=z?1,0?|z|??

9?n(6)ZT[2(u(n)?u(n?10))]=?2n?0?nz?n?1?2?10?1zz?10?11?2,0?|z|??

3.15 求以下序列的Z变换及收敛域,并在z平面上画出极零点分布图: (1)x(n)?RN(n),N?4 (2)x(n)?Ar

n??(3)x(n)??2N?n?0??ncos(?0n??)u(n),r?0.9,?0?0.5?rad,??0.25?rad

0?n?NN?1?n?2N,式中N?4

其它34?n?n解:(1) X(z)??Rn???(n)z??zn?02?4k?1?z1?z?4?1?z?1z(z?1)34,0?|z|??

z?1=0,零点为:zk?e4j,k?0,1,2,3;

z3(z?1)=0,极点为:z1,2?0,1

极零点分布图如题15解图(a)所示,图中z?1处的零极点相消。

(0n??)u(n)?(2) x(n)?Arcos?n12Ar[enj?0nej??e?j?0ne?j?]u(n)

X(z)?12??nj?0nA[?ren?0ej?z?n??rn?0?1ne?j?0ne?j?z?n]?12A[e1?rej?j?0z?1?e1?re?j??j?0z?1] =Acos??rcos(?0??)z(1?rej?0z?1)(1?re?j?0z?1),|z|?r

零点:z1?rcos(?0??)cos?,极点:z2?rej?0,z3?re?j?0

极零点分布图如题15解图(b)所示。 (3)令y(n)?R4(n),则

x(n?1)?y(n)*y(n)

zX(z)?[Y(z)],X(z)?z2?1[Y(z)]

2因为

Y(z)?1?z1?z?1?4?1?z?1z(z?1)234

因此得到 X(z)?z[z?1z(z?1)j2?443]?1z7[z?1z?14]

2极点为:z1?0,z2?1,零点为:zk?ek,k?0,1,2,3;

在z?1处的零极点相消,收敛域为:0?|z|??,极零点分布图如题15解图(c)所示。

题15解图

(a) (b) (c) 3.16 已知:

X(z)?1?312z?1?21?2z?1

求出对应X(z)的各种可能的序列表达式。

解:X(z)有两个极点:z1?0.5,z2?2,因为收敛域总是以极点为边界,因此收敛域有以下三种情况:

|z|?0.5,0.5?|z|?2,|z|?2

三种收敛域对应三种不同的原序列。

(1) 当收敛域为|z|?0.5时,由收敛域可得原序列为左边序列。

X(z)?1?3121?z?121?2z?1

查表3-2可得

x(n)??[3?()?2?2]u(?n?1)

2nn(2) 当收敛域为0.5?|z|?2时,

X(z)?1?312z?1?21?2z?1?X1(z)?X2(z)

由收敛域可得X1(z)对应的原序列为右边序列,而X2(z)对应的原序列为左边序列,查表3-2可得

x(n)?3?()nu(n)?2?2nu(?n?1)

21(3) 当收敛域为|z|?2时,由收敛域可得原序列为右边序列。

X(z)?1?3121?z?121?2z?1

查表3-2可得

x(n)?[3?()?2?2]u(n)

2nn3.17 已知x(n)?anu(n),(1)x(n)的Z变换; (2)nx(n)的Z变换;

0?a?1。分别求:

(3)a?nu(?n)的Z变换。 解:

?(1) X(z)?ZT[au(n)]?n?n???au(n)zn?n?11?az?1,|z|?a

(2) ZT[nx(n)]=?zddz0X(z)??az(1?az??1?1)2,|z|?a

(3) ZT[a?nu(?n)]=

?n???a?nz?n??n?0aznn?11?az,|z|?a?1

3.18 已知X(z)??3z2?5z?1?1?2z?2,分别求:

(1)收敛域0.5?|z|?2对应的原序列x(n); (2)收敛域|z|?2对应的原序列x(n)。

解:X(z)有两个极点:z1?0.5,z2?2,所以利用部分分式进行展开为:

X(z)?A11?2z?1?1?A212z?1

其中

?A1?(1?2z?132z?1)(1??1232?(1?2zzz?1?1)|z?2??1

)?1A2?(1?2z?1)(1?12?(1?z?112z?1)|z?12?1

)所以

X(z)?X1(z)?X2(z)?A11?2z?1?1?A212z?1??11?2z?1?1?112z?1

(1)收敛域0.5?|z|?2对应的原序列x(n),由收敛域可得X1(z)对应的原序列为左边序列,而X2(z)对应的原序列为右边序列,查表3-2可得 x(n)?()u(n)?2u(?n?1)

21nn(2)收敛域|z|?2对应的原序列x(n),由收敛域可得X1(z)、X2(z)对应的原序列都为右边序列,查表3-2可得

x(n)?()u(n)?2u(n)

21nn3.19 分别用长除法、部分分式法求以下X(z)的反变换:

1?1314zz?1(1)X(z)?1?,?2|z|?12

(2)X(z)?1?2z1?14z?1,?2|z|?12

解:

(1)部分分式法:X(z)有两个极点:z1?0.5,z2??0.5,所以利用部分分式进行展开为:

1?X(z)?1?1314zz?1??2A11?12z?1?1?A212z?1

1?A1?(1?12z?113z?1)(1?13z?112?(1?z?112z?1)|z?0.5?16

)1?A2?(1?12z?1)(1?12?(1?z?112z?1)|z??0.5?56

)所以

1?X(z)?1?1314zz?11?612?z?15612

z?1?21?1?由收敛域|z|?12可得原序列为右边序列,查表3-2可得

x(n)?[11n51n()?(?)]u(n) 6262长除法

x(n)??1,???1111?,,?,?? 341216?(2)部分分式法:X(z)有两个极点:z1?0.5,z2??0.5,所以利用部分分式进行展开为:

X(z)?1?2z1?14z?1??2A11?12z?1?1?A212z?1

A1?(1?1?2z12z?1?1)(1?12?(1?z?112z?1)|z?0.5??32

)A2?(1?1?2z12z?1?1)(1?12?(1?z?112z?1)|z??0.5?52

)所以

1?2z1?14z?1??1?1232z?15?1?212X(z)? z?1?2由收敛域|z|?12可得原序列为左边序列,查表3-2可得

x(n)?[31n51n()?(?)]u(?n?1) 2222长除法

x(n)n?0 x(?1) x(?2) x(?3) x(?4) x(?5) … … 0 8 -4 32 -16 128 3.20 设确定性实序列x(n)的自相关函数用下式表示:

?rxx(m)??x(n)x(n?m)

n???试用x(n)的Z变换X(z)和傅里叶变换X(ej?)分别表示自相关函数的Z变换Rxx(z)和傅里叶变换Rxx(ej?)。

?解: rxx(m)??x(n)x(n?m)

n??????m??Rxx(z)???x(n)x(n?m)zm???n?????n???x(n)?x(n?m)zm????m

令m??n?m,则

??Rxx(z)??n????x(n)?x(m?)zm?????n?m??n

=

?n???x(n)z?x(m?)zm?????m??X(z?1)X(z)

?或者 rxx(m)??x(n)x(n?m)?n????1x(m)?x(?m)

Rxx(z)?X(z)X(z) Rxx(ej?)=Rxx(z)|z?ej??X(ej?)X(e?j?)j?

)=|X(ej?因为x(n)是实序列,X(e?j?)?X*(ej?),因此Rxx(e3.21 用Z变换法解下列差分方程:

(1)y(n)?0.9y(n?1)?0.05u(n),y(n)?0,n??1

)|。

2(2)y(n)?0.9y(n?1)?0.05u(n),y(?1)?1,y(n)?0,n??1 (3)y(n)?3y(n?1)?2y(n?2)?u(n),

y(?1)?0,y(?2)?12,y(n)?0,n??3

解:

(1) 对方程两边进行Z变换得

Y(z)?0.9Y(z)z?1?0.051?z?1

Y(z)?0.05(1?0.9z?1)(1?z?1)运用部分分式法得

Y(z)?0.05(1?0.9z?1)(1?z?1)??0.451?0.9z?1?0.51?z?1

由y(n)?0,n??1知,y(n)是因果序列,查表3-2得

y(n)?(?0.45?0.9?0.5)u(n)

n(2) 对方程两边进行Z变换得

?1Y(z)?0.9z[Y(z)??1?k???y(k)z?k]?0.051?z?1

Y(z)?0.9z[Y(z)?y(?1)z]?Y(z)?0.9zY(z)?0.9??1?10.051?z?1?1

0.051?z

Y(z)?0.95?0.9z(1?0.9z?1?1?1)(1?z)

当n?0时,运用部分分式法得 Y(z)?查表3-2得

y(n)?(0.45?0.9?0.5)u(n)

n0.95?0.9z(1?0.9z?1?1?1)(1?z)?0.451?0.9z?1?0.51?z?1

总结得到

y(n)?(0.45?0.9?0.5)u(n)??(n?1) (3) 对方程两边进行Z变换得

Y(z)?3z[Y(z)?y(?1)z]?2z?1?2n[Y(z)?y(?1)z?y(?2)z]?211?z?1

当n?0时,运用部分分式法得

Y(z)?z(1?3z?1?1?2?2z)(1?z?1)?z(1?z?1?1?1)(1?z)(1?2z?1)

1?1632?? ?1?1?11?z1?z1?2z1612n?2查表3-2得

y(n)?(?(?1)?23(?2))u(n)

n总结得到

y(n)?(16?12(?1)?n23(?2))u(n)?n12?(n?2)

3.22 设线性时不变系统的系统函数H(z)为

1?a?1H(z)?z?11?az?1,a为实数

(1) 在z平面上用几何法证明该系统是全通网络,即|H(ej?)|?常数。 (2) 参数a如何取值,才能使系统因果稳定?并画出其极零点分布图及收敛域。 解:

(1) H(z)?1?a?1z?11?az?1?z?a?1z?a,极点:a,零点:a?1

设a?0.6,极零点分布图如题22解图(a)所示。已知|H(ej?)|等于极点矢量的长度除以零点矢量的长度,由题22解图(a)得到

|H(ej?)|?H(z)|z?ej??OAOC?OBOA?1az?a?1z?a|z?ej??|ej??a?1ej??a|?ABACABAC

1a因为?AOB??AOC,所以|H(ej?,所以?AOB~?AOC,故?。

)|?ABAC?1a,故H(z)是一个全通网络。

(2)只有选择|a|?1才能使系统因果稳定。设a?0.6,极零点分布图及收敛域如题22解图(b)所示。

(a) (b)

题22解图

3.23 设系统由下面差分方程描述:

y(n)?y(n?1)?y(n?2)?x(n?1)

(1) 求系统的系统函数H(z),并画出极零点分布图。

(2) 限定系统是因果的,写出H(z)的收敛域,并求出其单位脉冲响应h(n);

(3) 限定系统是稳定的,写出H(z)的收敛域,并求出其单位脉冲响应h(n)。 解:

(1) y(n)?y(n?1)?y(n?2)?x(n?1) 对方程两边进行z变换,得

Y(z)?Y(z)z?1?Y(z)z?2?X(z)z?1

Y(z)X(z)z1?z?1因此 H(z)???1?z?2

零点:z0=0

1?251?25极点:令1?z?1?z?2=0,求出极点,z1=

题23解图 极零点分布图如题23解图所示。

,z2=

(2)限定系统是因果的,H(z)的收敛域必须包含?,即|z|?冲响应h(n),h(n)?IZT[H(z)]。

z1?z?11?25,求出其单位脉

H(z)??1?z?2?1?A11?25z?1?1?A21?252z5

z?1A1?(1?z1?25z?1?1)(1??11?25?(1?z?11??1)|z?1?25?15

)A2?(1?z1?25z?1)(1?1?25?(1?z?11?25z?1)|z?1?25??15

)因为h(n)是因果的,查表3-2得到

151?251?25h(n)?[()?(n)]u(n)

n(3)限定系统是稳定的,H(z)的收敛域必须包含单位圆,即

1?25?|z|?1?25,求出

其单位脉冲响应h(n),h(n)?IZT[H(z)]。由(2)的结论,

1 H(z)?z1?z?1?1?z?2?X1(z)?X2(z)?1?51?25z?1??1?1?1552z?1

由收敛域得X1(z)对应的原序列为左边序列,X2(z)对应的原序列为右边序列,查表3-2得到

h(n)??15(1?25)u(?n?1)?n15(1?25)u(n)

n3.24 已知线性因果网络用下面差分方程描述:

y(n)?0.9y(n?1)?x(n)?0.9x(n?1)

(1) 求网络的系统函数H(z)及单位脉冲响应h(n);

(2) 写出网络传输函数H(ej?)表达式,并定性画出其幅频特性曲线; (3) 设输入x(n)?e解:

(1) y(n)?0.9y(n?1)?x(n)?0.9x(n?1) 对方程两边进行z变换,得

Y(z)?0.9Y(z)z?1?X(z)?0.9X(z)z?1

Y(z)X(z)1?0.9z1?0.9z?1?1j?0n,求输出y(n)。

因此 H(z)??

h(n)?IZT[H(z)]

H(z)?(a)

1?0.9z1?0.9z?1?1??1?21?0.9z?1因为单位脉冲响应h(n)为因果序列,查表3-2得

h(n)???(n)?2?0.9u(n)

n(2) H(ej?)?H(z)|z?ej??1?0.9e1?0.9ej?j?

极点z1=0.9,零点z2=-0.9,极零点图如题24解图(a)所示,根据极零点分布定性画出的幅度特性

(b) 题24解图

如题24解图(b)所示。 (3) x(n)?ej?n

0 y(n)?ej?0nH(ej?0)?ej?0n1?0.9e1?0.9ej?0j?0

3.25 已知网络的输入和单位脉冲响应分别为x(n)?anu(n),h(n)?bnu(n),0?a?1,

0?b?1,试求:

(1)用卷积法求网络输出y(n); (2)用Z变换法求网络输出y(n)。 解:(1)用卷积法求网络输出y(n)

?y(n)?x(n)?h(n)??bm???mu(m)an?mu(n?m)

n =?bman?m

m?0 n?0,

nnm y(n)??bm?0an?m?an?bm?0ma?m?an1?a?n?1bn?11?ab?1?an?1?bn?1a?b

n?0,

y(n)=0

综上得到 y(n)?an?1?bn?1a?bu(n)

(2)用Z变换法求网络输出y(n)

X(z)?11?az?1,H(z)?11?bz?1

Y(z)?X(z)H(z)?1(1?az?1)(1?bz?1)

利用部分分式法展开得

aY(z)?1(1?az?1b)(1?bz?1)?a?bb?a ??1?11?az1?bz由题意知,该系统为因果系统,所以系统在n?0之后才有输出,查表3-2得

n?1y(n)?a?bn?1a?bu(n)

3.26 如果x1(n)和x2(n)是两个不同的因果稳定实序列,求证:

1???2????X1(ej)Xj?2(e)d??[1?2????X1(ej?)d?][12????X2(ej?)d?]

式中,X1(ej?)和X2(ej?)分别表示x1(n)和x2(n)的傅里叶变换。 解: FT[x1(n)*x2(n)]=X1(ej?)Xj?2(e) 对上式进行IFT,得 1??2????X1(ej)X?n2(ej?)ejd??x1(n)?x2(n)

令n?0

1?2????X1(ej?)X2(ej?)d??x1(n)?x2(n)|n?0 因为x1(n)和x2(n)都是因果稳定实序列,

n[x1(n)*x2(n)]|n?0??x1(m)x2(n?m)|n?0?x1(0)x2(0) m?0x1(0)x2(0)?[12???j???X1(e)d?][12?????X2(ej?)d? 联立(1)、(2)、(3)式,得

1???[1??2????X1(ej?)X2(ej?)d2????X1(ej?)d?][12????X2(ej?)d?]

3.27 如序列h(n)是因果序列,其傅里叶变换的实部如下式:

Hacos?R(ej?)?1?1?a2?2acos?,|a|?1

求序列h(n)及其傅里叶变换H(ej?)。

j?解: Hj??acos?1?0.5a(ej??e?)R(e)?11?a2?2acos??1?a2?a(ej??e?j?)

?11?a2H)R(z)?1?0.5a(z?z)1?a2?a(z?z?1)?0.5?0.5((1?az)(1?az?1)

对上式求IZT,得到序列h(n)的共轭对称序列he(n)。

2HR(z)?0.5?0.5(1?a)0.5(1?az)(1?az?1)?0.5?0.51?az?1?az?1

1)

2)

3)

( ( (

因为h(n)是因果序列,所以he(n)必为双边序列,所以收敛域为a?|z|?a?1,查表3-2得

?0.5a?n,n?0? he(n)??1,n?0?0.5an,n?1??0,n?0??1,n?0???nh(n)??he(n),n?0???a,n?0

?2h(n),n?0??0,n?0?e??所以 h(n)?anu(n)

?j?n?j?nH(e)??n?0ae?11?ae?j?

3.28 如序列h(n)是因果序列,h(0)?1,其傅里叶变换的虚部如下式:

HI(ej?)??asin?1?a?2acos?2,|a|?1

求序列h(n)及其傅里叶变换H(ej?)。

12j2j??j?解: HI(ej?)??asin?1?a?2acos??a(z?z2?1?a?(e?ej?))21?a?a(e12j?e?j?

HI(z)?12j?)?11?a?a(z?z)???a(z?z?1)?1(1?az)(1?az)

jHI(z)对应h(n)的共轭对称反序列ho(n),因此jHI(z)的反变换就是ho(n)。

?a(z?z?1jHI(z)?0.5?)?1(1?az)(1?az)??0.51?az?0.51?az?1]

因为h(n)是因果序列,所以ho(n)必为双边序列,所以收敛域为a?|z|?a??0.5a?n,n?0? h0(n)??0,n?0?0.5an,n?0??1,查表3-2得

?1,n?0??n?h(n)?ho(n)u?(n)?h(0)?(n)??a,n?0?

?0,n?0???所以 h(n)?au(n)

n?H(ej?)??n?0aen?j?n?11?ae?j?

第四章

?jk?j~34.4 X(k)=1?e?e?2?3k?e?j?k, k?0,1,2,3,4,5

4.5 周期卷积为{14,12,10,8,6,10}

1?e?j2?N2?NNk4.7 (1)X(k)=

1?e?jk?N,k?0 ???0,k?1,2,3,...,N?1(2) X(k)=1 (3) X(k)=Wn(4) X(k)=

kn0,k?0,1,2,...,N?1

kmkN1?WN1?W,k?0,1,2,...,N?1

(5) X(k)=

1?e1?e?j2?(m?k)2?N?j(m?k)?N,k?m??,k?0,1,2,...,N?1 ?0,k?m?N?,k?m,k?N?m(6) X(k)=?2,k?0,1,2,...,N?1

?0,k?m,k?N?m?(7) X(k)=

1?e1?ej?0N2?N,k?0,1,2,...,N?1

k)j(?0??j?N?j?N1?1?e01?e0?(8) X(k)=2?2??j(??k)?j(?0?k)2j0NN1?e?1?e?j?N?j?N1?1?e01?e0?(9) X(k)= 2?2?j(?0?k)?j(?0?k)2?NN1?e?1?e?N(N?1),k?0?2?(10) X(k)=?

?N?,k?1,2,3,...,N?1k??1?WN?? ???? ??

4.8 (1) x(n)?cos( (2) x(n)?sin(2?Nmn??),n?0,1,2,....,N?1

2?Nmn??),n?0,1,2,....,N?1

kk?rX(), ?整数??rr4.11 H(k)??,0?k?rN?1

k?0, ?整数 ?r?4.14 n=5到n=14的点。

4.16 (1)X(z)?1?z1?z?N?1, 极点:z1?0,(N?1)阶极点z2?1j2?Nk 零点:zk?e,k?0,1,2,....,N?1.

(2)X(ej?)?1?e?j?N?j?

?j2?k2?N1?eNkk (3)X(k)?1?WN1?WN?1?e1?ejk?N,k?0???0,k?0

第五章

5.1 直接计算所需时间为6.290432 s,FFT计算需要时间为35.84ms;信号最高频率为6666.7Hz。

5.2 (1)9965段;(2)5102.08kHz;(3)2550kHz。 5.6 15.625 Hz.

5.7 (1)0.1s; (2)5kHz; (3)1024

第六章

6.1 分别用直接型、级联型和并联型实现下面的传递函数,并画出流图 (1)H(z)?3z?3.5z?2.5z(z?z?1)(z?0.5)3232

2(2)H(z)?4z?2.828z?z(z?1.414z?1)(z?0.7071)322

解:(1)直接型:H(z)?3z?3.5z?2.5z(z?z?1)(z?0.5)2=

3?3.5z1?1.5z?1?1?2.5z?2?2?3?1.5z?0.5z

级联型:H(z)?3?3.5z1?z21?0.5z?1?1?2.5z?z?2?21 ??11?0.5z?1?2并联型:H(z)??1?1?z1?z?1?z

直接型、级联型、并联型结构分别如图p6.1(a),(b),(c)所示 x?n?1.5?1.53y?n?x?n?3y?n?z?1?3.5z?12.5z?1?1z?1?3.5z?12.50.5z?10.5(a)直接II型x?n?0.5(b)级联型2z?1y?n?z?1?1?1z?1(c)并联型图p6.1 IIR的网络结构

(2)直接型:H(z)?4z?2.8284z?z(z?1.4142z?1)(z?0.7071)?1322?4?2.8284z1?0.7071z?1?1?z?2?3?0.7071z

级联型: H(z)?4?2.8284z1?1.4142z2?z?z?2?1?21 ??11?0.7071z?1?2并联型:H(z)?1?0.7071z?1?2?1.4142z1?1.4142z?1?z

直接型、级联型、并联型结构分别如图p6.2 (a),(b),(c)所示

x?n?4?1z2.82840.7071?z?1y?n?x?n?1.41424?1z?2.8284?0.7071z?1y?n?z?11?0.7071z?1(a)直接II型x?n??0.70712(b)级联型2z?1y?n?z?11.4142?1.4142?1?1z(c)并联型图p6.2 IIR网络结构的信号流图

6.2 用横截型和级联型结构实现传递函数H(z)??1?1.4142z?1?z?2??1?z?1?。 解:

横截型:H(z)??1?1.4142z?1?z?2??1?z?1??H(z)?1?1.4142z?1?0.4142z?2?z?3 级联型:H(z)??1?1.4142z?1?z?2??1?z?1?

横截型结构如图p6.3(a)或(b)所示。级联型结构如图p6.3(c)所示。

x?n?1z?1z?1z?1?0.4142?0.41421x?n??1z?0.4142z?1y?n?(a)横截型z?1x?n?1y?n?z?1z?1z?1?0.4142?0.41421(c)级联型y?n?(b)横截型图p6.3 FIR网络结构信号流图

6.3 用直接I型及直接II型结构实现以下传递函数: H(z)??5?2z1?3z?1?1?0.5z?2?2?3?3z?z

解: 直接I型及直接II型结构如图p6.4(a),(b)所示

-5x?n?-5-3-3-1(a)直接I型图p6.4

z?12z?1z?1z?1y?n?x?n?-3-3-1z?12y?n?z?1-0.5z?1-0.5z?1

(b)直接II型6.4 用级联型结构及并联型结构实现以下传递函数 H(z)?3z?3.5z?2.5z32?z2?z?1??z?0.5?32

解 H(z)?3z?3.5z?2.5z?z2?z?1??z?0.5??1?3?3.5z?1?2.5z?2?1?z?2?z?1?1??1?0.5z?1?

??5z?z?2?1?3??0.5z?1?1??1?z?1??1?0.5z??21?0.5z?1?1?zz?2?z?1?1?3?3.5z1?1.5z?1?1?2.5z?2?2?3?1.5z?0.5z

级联型结构及并联型结构如图p 6.5 (a),(b)所示

3x?n?x?n?0.5z?12y?n?1-1z?1-3.5?1z2.50.5z?1y?n?-11z?1z?1

(a)级联型图p 6.5

(b)并联型6.5 设滤波器差分方程为y(n)?x(n)?13x(n?1)?34y(n?1)?18y(n?2),用直接I型、

II型以及全部一阶节的级联型、并联型结构实现它。

?解 Y(z)?X(z)13?1zX(z?)34?1z1Y(?)z8?2z Y()z3?11?2?1?1???Y(z)?1?z?z??X(z)?1?z?

483????H(z)?Y(z)X(z)1??1?3z13?1z??11??z?213z?11244824直接I型、II型以及全部一阶节的级联型、并联型结构如图p6.6(a)(b)(c)(d)所示。

(1?1z)(1??11?3101?1z?1??731?1z?1

z)?1x?n?z?11334z?1y?n?x?n?34z13z?1103?1y?n?18z?118(a)(b)z?1?73x?n?12z?11314z?1y?n?x?n?12y?n?14z?1(c)图p6.6

(d)

??0.2n?????n????6.6 已知滤波器单位脉冲响应为h(n)??,求横截型结构。

??0????????其它??解H(z)?1?0.2z?1?0.22z?2?0.23z?3?0.24z?4?0.55z?5 横截型结构如图p6.7

x(n)z?1z?1z?1z?1z?110.210.220.23图p6.7

0.240.25y(n)

6.7 用横截型和级联型结构实现传递函数H(z)?(1?1.4142z?1.41z42?z解 H(z)?(1?1?2?1?z?2)(1?z)。

?2?3?1?)(z1?1?1).41z42??1?10.4z14?2z

y(n)?x(n)?0.4142x(n?1)?0.4142x(n?2)?x(n?3)

横截型和级联型结构如图p6.8(a)(b)(c)所示。

z?1z?1z?11x(n)z?1z?1x(n)(a)1?0.4142?0.4142y(n)1?0.4142?b?x(n)z?1?0.4142z?1z?1y(n)?c?图p6.8

6.8 已知系统的状态方程和输出方程为

?w1(n?1)??2????????????(n)??4?????????x(n) ????w2(n?1)??1?????????w2(n)??0????(n)?y(n)??1?????????x(n)

?w2(n)?试根据其状态方程与输出方程画出网络结构图。 解 由状态方程与输出方程

w1(n?1)?4x(n)?2w1(n)?0.5w2(n) w2(n?1)?w1(n) y(n)?w1(n)+x(n)

网络结构如图p6.9所示。

x(n)42y(n)z?1w1z?1w2

0.5图p6.9

6.9 设FIR滤波器由系统函数H(z)?1?16.0625z形式、线性相位形式和级联形式的结构。

?4?z给定,求出并画出它的直接

?8解:

a. 直接形式。差分方程为y(n)?x(n)?16.0625x(n?4)?x(n?8)。直接形式结

构如图p6.10(a)所示。

b. 线性相位形式。差分方程可写为

所得结构如图p6.10(b)所示。 c. 级联形式。要用matlab协助 b=[1, 0, 0, 0, 16.0625, 0, 0, 0, 1]; [sos,G]=tf2sos(b,1) 程序运行的结果为:

? 1.0000 2.8284 4.0000 1.0000 0 0??? 1.0000 -2.8284 4.0000 1.0000 0 0? sos = ?? 1.0000 0.7071 0.2500 1.0000 0 0??? 1.0000 -0.7071 0.2500 1.0000 0 0??y(n)??x(n)?x(n?8)??16.0625x(n?4),

G = 1

其传递函数应写成

H(z)?(1?2.83z?1?4z)(1?2.83z?2?1?4z)(1?0.71z?2?1?0.25z)(1?0.71z?2?1?0.25z)

?2级联形式结构如图p6.10(c)所示。

x(n)x(n)z?4z?416.0625z?4z?416.0625y(n)(b)线性相位形式y(n)(a)直接形式x(n)z?1z?12.8284z?10.7071z?10.25z?1?0.7071z?10.25z?1?2.8284z?1y(n)4.04.0

(c)级联形式图p6.10

6.10 设

H(z)?1?z5?5?1?1??0.809?0.809z0.309?0.309z1? ?0.0848??0.5818?1?2?1?2?1?1?1.618z?z1?1.618z?z1?z????.画出频率样本结构图。

解: 频率样本形式如图p6.11所示。由图可见,当采样点数很大时,显然其结构很复杂,需要的乘法器和延时单元多。但对于窄带滤波器,大部分频率样本值H(k)为零,使二阶网络个数大大减少,所以频率样本结构适用于窄带滤波器。

?0.8090.6180.5818z?1z?10.809x(n)0.2?10.3090.0848y(n)?z?51.618?1z?1z?1?0.30911图p6.11 频率采样结构

z?1

第七章

7.1 设某IIR低通滤波器的设计指标如下:

1)通带截止角频率?c?0.1?rad 2)通带最大衰减?p?3dB 3)阻带起始角频率?s?0.25?rad 4)阻带最小衰减?s?15dB

试由冲激不变法设计该滤波器。

??解: 相应归一化滤波器的阻带起始角频率?s?s?c?0.25?0.1??2.5

?和阻带最小衰减?,查巴特沃思滤波器阻带衰减曲线簇,可得满足要求的的最低阶次由?ssN?2。查表可得

?(s?)? Ga1??s?t?22??12s

?a(t?)? g2e?sint?2?ua(t?)

?a(t?)|t??n??g?a(t?)|t??0.1?nh(n)?gs

?2e?0.1?2n?sin0.1?n2

?u(n)H(z)??c?ZT[h(n)]2e

?0.1???0.1?2?sin?0.1?20.1?2?z?1?0.1?21?2?cos?0.1?2

?2?e?1?z?1?e?z?20.0783873z1?1.5622441z?1?0.6412805z?27.2 一数字滤波器采样频率fs = 1kHz,要求滤除100Hz的干扰,其3dB的边界频率为95Hz

和105Hz,原型归一化低通滤波器为

w1=95/500; w2=105/500;

[B,A]=butter(1,[w1, w2],'stop'); [h,w]=freqz(B,A); f=w/pi*500;

plot(f,20*log10(abs(h))); axis([50,150,-30,10]); grid;

xlabel('频率/Hz') ylabel('幅度/dB')

图7-32 巴特沃思滤波器

7.3 设计一数字高通滤波器,它的通带为400~500Hz,通带内容许有0.5dB的波动,阻带内衰减在小于317Hz的频带内至少为19dB,采样频率为1,000Hz。

wc=2*1000*tan(2*pi*400/(2*1000));

wt=2*1000*tan(2*pi*317/(2*1000)); [N,wn]=cheb1ord(wc,wt,0.5,19,'s'); [B,A]=cheby1(N,0.5,wn,'high','s'); [num,den]=bilinear(B,A,1000); [h,w]=freqz(num,den); f=w/pi*500;

plot(f,20*log10(abs(h))); axis([0,500,-80,10]); grid; xlabel('') ylabel('幅度/dB')

图7-33 巴特沃思高通滤波器

7.4试设计一巴沃特什低通滤波器,要求在20rad/s处的幅频响应衰减不多于-2dB;在30rad/s处的衰减大于-10dB。 解 技术指标为

?p?20, ?p=-2dB,?p?30,?s=-10dB 将上述参数代入式(7.2.6)后可得 N?lg[(100.2?1)/(10?1)]12lg(20/30)?3.371

则选N=4,将此N=4代入式(7.2.7)可得

?c?20/(100.2?1)1/8?21.387

查表可得4阶归一化巴特沃思低通滤波(?c?1)的传递函数为 H4(s)?1(1?0.765s?s)(1?1.848s?s)22

当?c=21.387时,用s/?c置换上式中的s后,并化简可得

H4(s)|?0.209?10262s?s21.387(457.4?16.37s?s)(457.4?39.52s?s)

7.5 用冲激响应不变法设计一个满足以下技术指标要求的巴特沃思低通滤波:幅度响应在通带截止频率?p?0.2613?处的衰减不大于0.75dB,在阻止截止频率?s?0.4018?处的衰减不小于20dB。求巴特沃思模拟低通滤波的传输函数Ha(s)。 解 (1) 求滤波器的阶数N

lg[N?10100.1ap0.1as?1?1)1]2?lg[100.1?0.750.1?20?11lg(?p?s10?10.2613?lg()0.4018?]2?lg[0.1885199lg0.6503]2??1.3602?0.1869?7.2777

取N=8。

(2)求滤波器的3dB截止频率?c

?p[100.1ap?1]?12N??c??s[10?12N0.1as?1]?12N ?1]?其中?P[10?s[100.1ap?1]?1]=0.2613?[100.1?0.75?12?8=0.9111

0.1as?12N=0.4018?[100.1?20?1]12?8=0.9472

因此 0.9111??c?0.9472

选取?c=0.9111,准确满足通带指标要求,超过阻带指标要求。

(3) 求Ha(s)的极点 sk??cej(?2N?k?N??2)?0.911e19??j(?16k)8,k=0,1,…,15

其中,左半s平面的极点为 s0=0.9111e s1=0.9111e s2=0.9111e s3=0.9111e*3j916?=-0.1778+j0.8936

?8)j(916??=-0.5062+j0.7575 =-0.7575+j0.5062 =-0.8963+j0.1778

3?)8j(916???4)j(916??3?)8916 s4=s=0.9111e*2?j(??=-0.8936-j0.1778

) s5=s=0.9111e s6=s=0.9111e s7=s=0.9111e(4) 求传输函数Ha(s) Ha(s)??cN2?1*kN*0*1?j(916???4=-0.7575-j0.5062 =-0.5062-j0.7575

?j(916???8)?j916?=-0.1778-j0.8963

?0.91113k?08?*0.47483k?0

*k?0?(s?sk)(s?s)?(s?sk)(s?sk)?[s?(s?sk)s?sksk]2*=

0.4748(s?0.3556s?0.83)(s?1.0124s?0.83)(s?1.515s?0.83)(s?1.7872s?0.83)0.4748s?4.67s?10.905s?16.536s?17.7s?13.715s?7.515s?2.671s?0.47587654322222

7.6 设计一切比雪夫高通滤波器,采样频率fs?2.4kHz,在频率fp?160Hz处衰减不大于?p?3dB,在频率fT?40Hz处衰减不小于?T?48dB. 将高通数字滤波器的频率指标换算成数字频率 ?p?T?P?2?fpfs?2?15

?s?T?T?2?fTfs??30

Wp = input('Normalized passband edge = '); Ws = input('Normalized stopband edge = '); Rp = input('Passband ripple in dB = ');

Rs = input('Minimum stopband attenuation in dB = '); [N,Wn] = cheb1ord(Wp,Ws,Rp,Rs)

[b,a] = cheby1(N,Rp,Wn,'high'); [h,omega] = freqz(b,a,256);

plot (omega/pi,20*log10(abs(h)));grid; xlabel('\\omega/\\pi'); ylabel('Gain, dB'); title('Type I Chebyshev Highpass Filter'); Normalized passband edge = 0.133

Normalized stopband edge = 0.033 Passband ripple in dB = 3

Minimum stopband attenuation in dB = 48 结果是 N =

3 Wn =

0.1330

滤波器最小阶数是3,截止频率为0.133?。

图7-34 切比雪肤高通滤波器

7.7 设计一个数字带阻滤波器,通带下限频率?l=0.19?rad,阻带下截止频率?sl=0.198?rad ,阻带上截止频?sh=0.202?rad率,通带上限频率?3=0.21?rad,阻带

最小衰减?s=13dB,?l和?3处衰减?p=3dB。采用巴特沃斯型。 解

(1) 数字带阻滤波器技术指标:

?l=0.1?9?sl=0.19?8rad ? 3 ?=0.21rad? s h r?ad =3dBp

=?0.202?rsad =13dB(2) 模拟带阻滤波器的技术指标:

设T=1,则有

?l?2tan?s1?2tan12?l?0.615rad/s,?u?2tan12?u?0.685rad/s1212

?s2?0.685rad/s?s1?0.615rad/s,?s2?2tan阻带中心频率平方为

2?.42 1 ?2??1?30阻带带宽为

B??3??1?0.07rad/s 将以上边界频率对B归一化:

?l=8.786?3=9.786 ?sl=9.186,?sh=9.386

?2=?l?3=85.982 (3) 模拟归一化低通滤波器的技术指标:

λp=1,αp=3dB

?s??sh?2sh??22?4.434,?s?13dB

(4) 设计模拟低通滤波器: ?sp??s?p?4.434?0.1?pN?log[(10?1)/(10?0.1?s

?1)]?0.992log1/?sp5) 将G(p)转换成模拟阻带滤波器Ha(s): p?sBs??222

Ha(s)?G(p)p?sBs??222(6) 将Ha(s)通过双线性变换,得到数字带阻滤波器H(z)。

sBs??022p?s?21?z?1?2(1?z?12?2)B?121?z?14(1?z)??0(1?z)2

2(1?z?2)BH(z)?G(p)p?4(1?z??120.969(1?619z1?1.569z?1?1?z?2)?12)2??0(1?z)?0.939z?27.8 使用双线性变换法设计一个切比雪肤数字低通滤波器,假设取样频率为10KHz,通带截止频率fp?1KHz处衰减不大于?p?1.8dB,在阻带截止频率fs?1.5KHz处衰减不小于?s?12dB。

解 (1). 将fP和fs转换成数字频率?P和?s

?P?2?fP?2??1000?2000?

??150?0 ?s?2?fs?23?00 0 T?1f?110?10?43?10

?4?P?T?P?10?2000??0.2?

?s?T?s?10?4?3000??0.3?

(2)取T=1,将数字频率?P和?T预畸变,得预畸变后的?P和?s

(3)计算滤波器通带波纹参数?,?由允许的通带波纹确定。因为在?P上幅度响应衰减不大于aP?1.8dB故

??10ap10?1?0.1?s?1100.1?1.8?1?0.7166

chN??1?10?1??100.1?p?1?ch(?s?p)?2.3177

取N=3。

(5).求切比雪夫滤波器的极点 设极点为sk??k?j?k则计算可得

?k???csin(2k?1?N2N?2k?1?1?11?k??ccos()ch(ch) (k=1,2,3,…,2N)

N2N?1Nsh??1)sh(1sh?11)

令 a?sh(121211?1N), b?ch(11Nch?11?)

?13a?(aN?a)?0.5(3.11223?3.1122)?0.3875

1b?(aN?a?1N1)?0.5(3.11223?3.1122?13)?1.0725

取?C??P?0.6498,由上式的s平面左半平面的极点

s0??0.3875?0.6498sin(s1??0.3875?0.6498sin(*?2?3)?j0.6498cos(??2?3)??0.1259?j0.6035

?2?3?3)?j1.0725?0.6498cos(?2?3??3)??0.2518

s2?s0??0.1259?j0.6035

(6).用左半s平面的3个极点构成传输函数Ha(s),便得到一个稳定的模拟切比雪夫低通滤波器,即 Ha(s)?KD(s)?K(N?1)2k?1 (N为奇数)

*k(s?sP)?(s?sk)(s?s)*其中,sk是s平面左半平面的极点,sk是sk的共轭极点,sP是s平面左半平面实轴上的极点。

系数K由s=0时滤波器幅度响应的值Ha(0)确定,当N为奇数时, Ha(0)=1。 因此 K?0.2518(0.1259?j0.0635)(0.1259?j0.0635)?0.0957

0.0957(s?0.2518)(s?0.1259?j0.6035)(s?0.1259?j0.6035)0.0957(s?0.2518)(s?0.2518s?0.38)2Ha(s)??(7).用双线性法求数字滤波器的系统函数

在Ha(s)的表达式中,令 s?21?zT1?z0.0957(2(1?z)1?z?1?1?1?1,其中取T=1,得

H(z)??0.2518)((2?2z1?z?13?1?1)?0.2518(22?2z1?z?1

)?0.38)?1?0.0957(1?z)?110.99689(1?0.7764z)(1?1.4824z0.0957?0.0909(1?z)(1?0.7764z)(1?1.4824z?3?1?1?13?1?0.7938z)?2??0.7938z?2)

?8.6991?10(1?z)(1?0.7764z)(1?1.4824z?1?1?13?0.7938z?2)

7.9 用矩形窗设计一个线性相位高通滤波器

Hd(ej??e?j(???)????c????)??0??????c?0

(1)求出h(n)的表达式,确定α与N的关系。

(2)问有几种类型,分别是属于哪一种线性相位滤波器。 (3)若改用汉宁窗设计,求出h(n)的表达式。 解:(1)

?ej??hd(n)?12?12??e2?0Hd(ej?)ejn?d??12?12???????c??ce?j(???)?ejn?d??j???????c??cej(n??)?d??1j(n??)ej??ej(n??)????c???c2??ej(n??)?j(n??)?ej(n??)?c?e?j(n??)?c??ejn??(n??)sin?(n??)?c??(?1)n?c?Sa?(n??)?c?

hd(n)?ejn?sin[(n??)?c]?(n??)?hd(n)h(n)?hd(n)RN(n)???00?n?N?1其他

N?1为保证线性: ??2

(2)有两类适合实现高通滤波器:

hN为奇数,α为整数, d ( n ) 对α偶对称,即有

h ( n ) ? h ( N ? n ) , H ( ? ) ? H ( 2 ? ? ? ), 为第一种类型线性相位滤波器。 ? 1N为偶数,α不为整数, h d ( n ) 对α奇对称,为第四种类型线性相位滤波器 (3)

??2???n?ch(n)?hd(n)WN(n)?(?1)Sa?(n??)?c??1?cos?n??RN(n) ??N?1???

n?0,8 ? 0

? ? 0 . n?1,7022?? n=2,6?c??/4,N?9时,h(n)??0.1592 ??0.3842n?3,5?

n?4??0.5

?h(0)??h(7)?0??h(1)??h(6)??0.0443若?c??/4,N?8时,h(n)???h(2)??h(5)?0.2397?h(3)??h(4)??0.4631?

7.10 用海明窗设计一个线性相位正交变换网络,已知 ?j???je?????0?j? H(e)??d?j???0???? ??je

(1)求出h(n)的表达式,确定α与N的关系。

(2)N为奇数或是偶数对于h(n)的影响的主要差别是什么?那么应该选择N是偶数还是奇数?

(3)若用凯塞窗设计,写出h(n)的表达式。 解:(1)先不考虑延时,即对于

H'd(ej??j)????j?????00????

本文来源:https://www.bwwdw.com/article/6nnx.html

Top